What is the spectrum of the operator $T({x_1},{x_2},{x_3}, \ldots ) = ({x_1},\frac{{{x_2}}}{2},\frac{{{x_3}}}{3}, \ldots )$

functional-analysishilbert-spacesoperator-theoryspectral-theory

Let us define the operator $T$ over $\ell_2$
$$T({x_1},{x_2},{x_3}, \ldots ) = ({x_1},\frac{{{x_2}}}{2},\frac{{{x_2}}}{3}, \ldots )$$

what is the point spectrum of this operator and the spectrum?

attempt:

what is puzzling me that when I try to get the point spectrum by setting $T(x)=\lambda x$, I end up getting no solution since $\lambda x_n=x_n/n$ for all $n$. if I solve using $x_1$, I get 1, with $x_2$ I get $1/2$ and so on. Should I deduce the spectrum is empty?

Best Answer

For your additional question, consider the "canonical" base vectors of $\ell_2$. This gives you a lot of eigenvalues. Actually, your argument shows that there are no other eigenvalues. If you know a theorem about the spectrum of compact operators, you can now show that your operator is compact and are done. Otherwise, you have to calculate the resolvent.